Đến nội dung

Simpson Joe Donald nội dung

Có 290 mục bởi Simpson Joe Donald (Tìm giới hạn từ 26-04-2020)



Sắp theo                Sắp xếp  

#437591 Topic về Bất đẳng thức, cực trị THCS

Đã gửi bởi Simpson Joe Donald on 23-07-2013 - 20:53 trong Bất đẳng thức và cực trị



Cho a,b,c>0 .Chứng minh:$\sqrt{\frac{2a}{b+c}}+\sqrt{\frac{2b}{c+a}}+\sqrt{\frac{2c}{a+b}}\leq 3$

Ta sử dụng $ Cauchy-Schwarz$ dưới dạng:

$$\sqrt{Ax}+\sqrt{By}+\sqrt{Cz}\leq \sqrt{(A+B+C)(x+y+z)}\tag{1}.$$

Ta viết:

$$\sqrt{\frac{2a}{b+c}}+\sqrt{\frac{2b}{c+a}}+\sqrt{\frac{2c}{a+b}}=\frac{\sqrt{2a(c+a)(a+b)}+\sqrt{2b(b+c)(a+b)}+\sqrt{2c(b+c)(c+a)}}{\sqrt{(a+b)(b+c)(c+a)}}.$$

Áp dung $(1)$ với:

$$A=2a(c+a) \ \text{and} \ x=a+b \\ B=2b(a+b) \ \text{and} \ y=b+c \\ C=2c(b+c) \ \text{and} \ z=c+a $$





#442706 Topic về Bất đẳng thức, cực trị THCS

Đã gửi bởi Simpson Joe Donald on 14-08-2013 - 10:52 trong Bất đẳng thức và cực trị

 

Bài 3: Co a,b khác 0 thỏa mãn $2a^{2}+\frac{b^{2}}{4}+\frac{1}{a^{2}}=4$. Tìm min $S= ab+2014$

 

Bài 5: Cho x;y;z>0 ; x+y+z=1 . CMR $\sqrt{2x^{2}+xy+y^{2}}+\sqrt{2y^{2}+yz+z^{2}}+\sqrt{2z^{2}+zx+x^{2}}\geq \sqrt{5}$

 

Bài 3: Ta có:

$4=a^2+\dfrac{1}{a^2}-2+a^2+\dfrac{b^2}{4}+ab-ab+2 \\ =\left(a-\dfrac{1}{a}\right)^2+\left(a+\dfrac{b}{2}\right)^2-ab+2\ge -ab+2 \\ \implies ab\ge 2\implies S\ge 2016$

Bài 5 mình nghĩ đề là:

CMR $\sqrt{2x^{2}+xy+2y^{2}}+\sqrt{2y^{2}+yz+2z^{2}}+\sqrt{2z^{2}+zx+2x^{2}}\geq \sqrt{5}$




#491598 $\boxed{\text{Chuyên Đề}}$ Bất đẳng thức - Cực trị

Đã gửi bởi Simpson Joe Donald on 09-04-2014 - 10:51 trong Bất đẳng thức và cực trị

166)  Áp dụng bất đẳng thức Cauchy cho 2 số dương 
$\frac{1}{a.b.3}$+$\frac{1}{a^2+b^2+a.b}$ $\geq$ $\frac{4}{4.a.b+a^2+b^2}$=$\frac{4}{1+2.a.b}$  
​Áp dụng bất đẳng thức Cauchy cho 2 số dương : 
2.$\sqrt{a.b}$ $\leq$ $a+b$=1 
Suy ra: 4.a.b $\leq$ $(a+b)^2$=1 
Suy ra a.b $\leq$ 0,25 <=> a.b.2+1 $\leq$ 1,5 
Suy ra: $\frac{4}{1+2.a.b}$ $\geq$ $\frac{8}{3}$ 
=> $\frac{1}{a.b.3}$+$\frac{1}{a^2+b^2+a.b}$ $\geq$ $\frac{8}{3}$ <=> VT $\geq$ VP (dpcm)

 




#440966 $\boxed{\text{Chuyên Đề}}$ Phân tích đa thức thành nhân tử

Đã gửi bởi Simpson Joe Donald on 07-08-2013 - 09:52 trong Đại số

Phân tích các đa thức sau thành nhân tử:
$\fbox{1}. \ \ (x^2+y^2+z^2)^3+2(xy+yz+xz)^3-3(x^2+y^2+z^2) \\ \fbox{2}. \ \ x^9 -x^7-x^6-x^5+x^4+x^3+x^2-1 \\ \fbox{3}. \ \ x^9-x^7-x^6-x^5+x^4+x^3+x^2+x+1 \\ \fbox{4}. \ \ (x+y+z)^5-x^5-y^5-z^5 \\ \fbox{5}. \ \ bc(a+d)b-c) - ac(b+d)(a-c) + ab(c+d)(a-d)$

 




#440983 $\boxed{\text{Chuyên Đề}}$ Phân tích đa thức thành nhân tử

Đã gửi bởi Simpson Joe Donald on 07-08-2013 - 10:30 trong Đại số

Câu 2 ra : $(x+1)^{2}(x-1)^{3}(x^{4}+x^{3}+2x^{2}+x+1)!?$

Làm từng bước đi bạn :D




#491579 $\boxed{\text{Chuyên Đề}}$ Phương trình vô tỉ - Hệ phương...

Đã gửi bởi Simpson Joe Donald on 09-04-2014 - 09:50 trong Phương trình, hệ phương trình và bất phương trình

Một số bài tập tự luyện cho dạng bài trên:

 

Bài 79:

$7x^2+7x=\sqrt{\frac{4x+9}{28}}$

ĐK: $x \ge \frac{{ - 9}}{4}$
Đặt $\sqrt {\frac{{4x + 9}}{{28}}} = y + \frac{1}{2}\left( {y \ge \frac{{ - 1}}{2}} \right)$
Khi đó ta có hệ phương trình:
$\left\{ \begin{array}{l}
2x = 14{y^2} + 14y - 1(1)\\
2y = 14{x^2} + 14x - 1(2)
\end{array} \right.$
Ta có:
$\begin{array}{l}
\left( 1 \right) - \left( 2 \right) \Leftrightarrow 2\left( {x - y} \right) = 14\left( {{y^2} - {x^2}} \right) + 14\left( {y - x} \right)\\
\Leftrightarrow \left( {x - y} \right)\left( {14x + 14y + 16} \right) = 0 \Leftrightarrow \left[ \begin{array}{l}
x = y\\
x = \frac{{ - 8 - 7y}}{7}
\end{array} \right.
\end{array}$




#430945 $\boxed{Topic}$Ôn thi học sinh giỏi lớp 9 năm 2013-2014.

Đã gửi bởi Simpson Joe Donald on 27-06-2013 - 10:18 trong Chuyên đề toán THCS

 

$\boxed{2}$

a) Giải phương trình $\Leftrightarrow x(x-5)(x^2+3x+12)$$\Leftrightarrow x(x-5)(x^2+3x+12)$

$$x^2-x-2\sqrt{1+16x}=2$$

 

Bình phương rồi rút gọn ta được:

$x^4-2x^3-3x^2-60x=0$

$\Leftrightarrow x(x-5)(x^2+3x+12)=0$

$Leftrightarrow x=0 hoac x=5$
Thử lại: $\to x=5$ là nghiệm duy nhất của pt



#439514 Cauchy-Schwarz

Đã gửi bởi Simpson Joe Donald on 31-07-2013 - 19:29 trong Bất đẳng thức và cực trị

CMR:$3a^3+7b^3 \geq 9ab^2$   với mọi $a,b \geq 0$

$$\text{AM-GM:} \ \ 3a^3+7b^3 \geq 3(a^3+b^3+b^3) \geq 9 ab^2$$




#473724 Topic về Phương trình và hệ phương trình không mẫu mực

Đã gửi bởi Simpson Joe Donald on 29-12-2013 - 16:48 trong Phương trình, hệ phương trình và bất phương trình



Cac ban giai hộ phương trình này nhé :

$\sqrt{6x+10}=x^2-13x+2$

$PT\Leftrightarrow \sqrt{6x+10}=(x-4)^2-5x-14$ , (2)



Đặt : $\sqrt{6x+10}=y-4$



$(2)\Leftrightarrow \begin{cases}(y-4)^2=6x+10 , (3)\\(x-4)^2=y+5x+10 , (4)\end{cases}$



$(3)-(4)\Leftrightarrow (y-x)(x+y-7)=0\Leftrightarrow $ $\left[ \begin{matrix}x=y\\x=7-y\end{matrix}\right]$




#479408 Trận 2 - PT, HPT

Đã gửi bởi Simpson Joe Donald on 27-01-2014 - 16:34 trong Thi giải toán Marathon cấp THCS 2014

Cậu còn tai hại hơn tại sao $\frac{1}{4}=0$?

Mình nghĩ nó phải thế này

$0=3(x-\frac{1}{2})^{2}+\frac{1}{4}\geq \frac{1}{4}\Rightarrow 0\geq \frac{1}{4}$(vô lí)

ờ, mình gõ nhầm :P cái dấu bằng đó là dấu $>$




#479412 Trận 2 - PT, HPT

Đã gửi bởi Simpson Joe Donald on 27-01-2014 - 16:41 trong Thi giải toán Marathon cấp THCS 2014

ta có:

$8x^{2}+12y^{2}-20xy=0\Leftrightarrow 2x^{2}+3y^{3}-5xy=0\Leftrightarrow (x-y)(2x-3y)=0$                            (1)

từ (1) ta có hai trường hợp :

+) trường hợp 1: nếu x=y thay vào phương trình thứ hai ta có:

$4x^{2}-6x+1=x^{2}-3x\Leftrightarrow 3x^{2}-3x+1=0$

khi đó phương trình vô nghiệm

+)trường hợp 2: nếu $2x=3y$ 

thay vào phương trình đầu ta có:

$9y^{2}-9y+1=y^{2}-3y\leftrightarrow 8y^{2}-6y+1=0$

khi ấy phương trình có hai nghiệm là $x=\frac{1}{2}$ hoặc $x=\frac{1}{4}$

vậy phương trình có hai nghiệm là $\frac{1}{2}$ và$\frac{1}{4}$

Phương trình có 2 nghiệm???Nghiệm của hệ đâu???




#479404 Trận 2 - PT, HPT

Đã gửi bởi Simpson Joe Donald on 27-01-2014 - 16:29 trong Thi giải toán Marathon cấp THCS 2014

Ý của người viết có lẽ là để $0\geq \frac{1}{4}$ để dẫn đến mâu thuẫn

Tất nhiên là mình hiểu nhưng trong toán học không được viết như thế , dễ làm người đọc hiểu nhầm. Nếu mà như thế thì phải viết viết $3(x-\frac{1}{2})^2+\dfrac{1}{4}\ge \dfrac{1}{4}=0$( vô lý)




#479403 Trận 2 - PT, HPT

Đã gửi bởi Simpson Joe Donald on 27-01-2014 - 16:26 trong Thi giải toán Marathon cấp THCS 2014

$\left\{\begin{matrix} 8x^{2}+12y^{2}-20xy=0 & (1)\\ 4x^{2}-6x+1=y^{2}-3y& \end{matrix}\right.$

Xét (1)$\Leftrightarrow 8x^{2}+12y^{2}-8xy-12xy=0\Leftrightarrow 8x(x-y)-12y(x-y)=0\Leftrightarrow 4(x-y)(2x-3y)=0\Leftrightarrow$

Vậy hoặc x=y hoặc x=1,5y

với x=y ta thế vào $4x^{2}-6x+1=y^{2}-3y$$\Leftrightarrow 4x^{2}-6x+1=x^{2}-3x\Leftrightarrow 3x^{2}-3x+1=0$ vô nghiệm vì $\Delta$ nhỏ hơn 0

Với x=1,5y ta có $4x^{2}-6x+1=(1,5x)^{2}-4,5x\Leftrightarrow 1,75x^{2}-1,5x+1=0$ cũng vô nghiệm vì $\Delta$ nhỏ hơn 0

Vậy hệ phương trình vô nghiệm thực

Với $x=1,5y$ thì $4(1,5y)^2-6.1,5y+1=y^2-3y$ chứ nhỉ?? Bạn nhầm $x=1,5y$ với $y=1,5x$ rồi :D




#478892 Trận 2 - PT, HPT

Đã gửi bởi Simpson Joe Donald on 25-01-2014 - 07:01 trong Thi giải toán Marathon cấp THCS 2014



Rất xin lỗi các toán thủ đã vì post đề chậm trễ, sau đây là đề thi trận 2 MSS:

Đề của toán thủ : Best Friend

 

$$\left\{\begin{matrix} 8x^{2}+12y^{2}-20xy=0 & & \\ 4x^{2}-6x+1=y^{2}-3y & & \end{matrix}\right.$$

Thời gian làm bài tính từ: 23h ngày 24/1/2014

Lời giải của MSS 33

 

$$\left\{\begin{matrix} 8x^{2}+12y^{2}-20xy=0 \ \ \ (1)& & \\ 4x^{2}-6x+1=y^{2}-3y \ \ \ (2)& & \end{matrix}\right.$$

Nhận thấy $x=0;y=0$ không phải nghiệm của phương trình. Chia cả 2 vế phương trình $(1)$ cho $y^2$ ta được:

$$8\left(\frac{x}{y}\right)^2-20\frac{x}{y}-12=0\iff 2\left(\frac{x}{y}\right)^2-5\frac{x}{y}-3=0 \ \ \ (\star)$$

Đặt $\frac{x}{y}=a$, phương trình $(\star)$ trở thành:

$$2a^2-5a+3=0\iff \begin{bmatrix}a=1 & \\ a=\dfrac{3}{2}& \end{bmatrix}\implies \begin{bmatrix}x=y & \\ x=\dfrac{3y}{2}& \end{bmatrix}$$

$TH_1: \ \ x=y$, Thay vào phương trình $(2)$ ta được:

$$4x^2-6x+1=x^2-3x\iff 3x^2-3x+1=0$$

Phương trình trên vô nghiệm do $\Delta =-3<0$

$TH_2: \ \ x=\dfrac{3}{2}$, thay vào phương trình $(2)$ ta được:

$$4.\left(\frac{3y}{2}\right)^2-6.\frac{3y}{2}+1=y^2-3y\iff 8y^2-6y+1=0 \iff \begin{bmatrix}y=\frac{1}{2}& \\ y=\dfrac{1}{4}& \end{bmatrix}$$

Với $y=\dfrac{1}{2}\implies x=\dfrac{3.\dfrac{1}{2}}{2}=\dfrac{3}{4}$

Với $y=\dfrac{1}{4}\implies x=\dfrac{3.\dfrac{1}{4}}{2}=\dfrac{3}{8}$

Thử lại: Thoả mãn cả 2 nghiệm.

Vậy tập nghiệm của hệ phương trình là : $(x;y)=\left(\frac{3}{4};\frac{1}{2}\right);\left(\frac{3}{8};\frac{1}{4}\right)$

________________________________
$d = 10$
$S = 44$




#479401 Trận 2 - PT, HPT

Đã gửi bởi Simpson Joe Donald on 27-01-2014 - 16:21 trong Thi giải toán Marathon cấp THCS 2014

MSS19
$8x^{2}+12y^{2}-20xy=0 \Leftrightarrow 2x^{2}+3y^{2}-5xy=0 \Leftrightarrow 2x^{2}-2xy-3xy+3y^{2}=0 \Leftrightarrow 2x(x-y)-3y(x-y)=0 \Leftrightarrow (2x-3y)(x-y)=0 \Rightarrow 2x-3y=0$
hoặc $x-y=0$
Nếu $x-y=0$ thì $x=y$ mà $4x^{2}-6x+1=y^{2}-3y \Rightarrow 4x^{2}-6x+1=x^{2}-3x \Leftrightarrow 3x^{2}-3x+1=0 \Leftrightarrow 3x^{2}-3x+ \frac{3}{4} + \frac{1}{4} =0 \Leftrightarrow 3(x^{2}-x+ \frac{1}{4}) + \frac{1}{4} =0 \Leftrightarrow 3(x- \frac{1}{2})^{2} + \frac{1}{4} = 0 \geq \frac{1}{4} \Rightarrow$

vô lý
Nếu $2x-3y=0$ thì $x= \frac{3y}{2}$ mà $4x^{2}-6x+1=y^{2}-3y \Rightarrow 4.( \frac{3y}{2})^{2}-6. \frac{3y}{2}+1=y^{2}-3y \Leftrightarrow 9y^{2}-9y+1=y^{2}-3y \Leftrightarrow 8y^{2}-6y+1 =0 \Leftrightarrow 8y^{2}-4y-2y+1=0 \Leftrightarrow 4y(2y-1)-(2y-1)=0 \Leftrightarrow (4y-1)(2y-1)=0 \Rightarrow 4y-1=0$ hoặc $2y-1=0 \Rightarrow y= \frac{1}{4}$
hoặc $y= \frac{1}{2}$

mà $x= \frac{3y}{2} \Rightarrow$

Với $y= \frac{1}{4}$

thì $x= \frac{3}{8}$

Với $y= \frac{1}{2}$

thì $x= \frac{3}{4}$

Vậy nghiệm của hệ phương trình là $x= \frac{3}{8};y= \frac{1}{4}$

và $x= \frac{3}{4};y= \frac{1}{2}$
 

Đầu tiên, đoạn bôi đỏ đầu tiên , bạn viết $3(x- \frac{1}{2})^{2} + \frac{1}{4} = 0 \geq \frac{1}{4}$ , cái này dễ làm người đọc hiểu nhầm là $0\ge \dfrac{1}{4}????$

Đoạn thứ 2 có vẻ hơi khó hiểu nhỉ, cái chữ "hoặc" tự nhiên nhảy ra giữa dòng là như thế nào??? Có lẽ đoạn này lỗi latex???




#473133 Topic về Phương trình

Đã gửi bởi Simpson Joe Donald on 26-12-2013 - 23:03 trong Phương trình, hệ phương trình và bất phương trình

Có bài sau nhờ mn làm giúp:

 

Giải pt sau: $x^6+x^5+x^4+x^3+x^2=x+1.$

$$pt\iff x^6+x^4+x^3+x^5+x^3+x^2=x^3+x+1 \\ \iff (x^3+x+1)(x^3+x^2-1)=0$$




#491048 [TOPIC] Bài toán tính tổng các dãy số có quy luật

Đã gửi bởi Simpson Joe Donald on 06-04-2014 - 15:01 trong Đại số

 

Mình úp thêm một số bài toán tính tổng, các bạn tha hồ giải nhé  :luoi:

 

$\boxed{13}$ Chứng minh rằng $A=1^5 + 2^5 + 3^5 + 4^5 + ... + n^5 = \dfrac{1}{2}n^2(n+1)^2(2n^2+2n-1)$

 

$\boxed{14}$ Chứng minh rằng $A=1^3+2^3+3^3+....+n^3 = (1+2+3+4+...+n)^2$

 

$\boxed{15}$ Tính tổng: $A=1^k + 2^k + 3^k + 4^k + ... + n^k$ $(k > 0 )$

 

$\boxed{16}$ Tính tổng: $A= 1! + 2.(2!) + 3.(3!) + 4.(4!) + 5.(5!) + ... + n.(n!)$ với ($n! = 1.2.3.4.5. .... n$)

 

$\boxed{17}$ Tính tổng: $A= 1 + 2p + 3p^2 + ... (n+1)p^n$ $(p \neq 1)$

 

$\boxed{18}$ Chứng minh công thức: $k(k+1)(k+2)(k+3) - (k-1)k(k+1)(k+2)= 4k(k+1)(k+2)$. 

 

$\boxed{19}$ Tìm $x$: 

 $a,$ $(x+1) + (x+2) + (x+3) + ...+ (x+100) = 5070$

 $b,$ $1+2+3+4+... + x = 820$

 $c,$ $1 + \frac{1}{3} + \frac{1}{6} + \frac{1}{{10}} + ...... + \frac{2}{{x(x + 1)}} = \frac{{1991 + 1989}}{{1991}}$

 

$\boxed{20}$ Tính tổng: $A=\frac{1}{{{3^0}}} + \frac{1}{{{3^1}}} + \frac{1}{{{3^2}}} + ..... + \frac{1}{{{3^{2005}}}}$

 

$\boxed{21}$ Tính tổng: $A=1.2 + 2.5 + 3.8 + ... + n(3n-1)$

 

$\boxed{22}$ Tính tổng: $A=1 + p + p^2 + p^3+ p^4 + ... + p^n$ $(p \neq 1)$

 

$\boxed{23}$ Tính tổng: $A = \frac{3}{{{{(1.2)}^2}}} + \frac{5}{{{{(2.3)}^2}}} + ....... + \frac{{2n + 1}}{{{{\left[ {n(n + 1)} \right]}^2}}}$

 

$\boxed{23}$ Tính tổng: $ A = \frac{1}{{1.2.3.4}} + \frac{1}{{2.3.4.5}} + ...... + \frac{1}{{n(n + 1)(n + 2)(n + 3)}}$

 

22) 
$A.p=p+p^2+...........+p^{n+1}$ 
$A.(p-1)=p^{n+1}-1$ 
=> $A=\frac{p^{n+1}-1}{p-1}$




#485767 Trận 4 - Bất đẳng thức

Đã gửi bởi Simpson Joe Donald on 04-03-2014 - 11:19 trong Thi giải toán Marathon cấp THCS 2014

MSS-01: Nguyễn Đức Thuận

 

Từ bất đẳng thức (BĐT) AM-GM, ta có:

$(x+y)^2\geq 4xy(1)$

$x^2+y^2\geqslant \frac{(x+y)^2}{2}(2)$

 

$(1)\Rightarrow (x+y)^3+(x+y)^2\geq 2\Leftrightarrow a^3+a^2-2\geqslant 0$ (a=x+y)

$\Rightarrow (a-1)(a^2+2a+2)\geqslant 0\Leftrightarrow a\geq 1$ (Do $a^2+2a+2=(a+1)^2+1>0$ với mọi a)

$(2)\Rightarrow t=x^2+y^2\geq \frac{a^2}{2}\geq \frac{1}{2}$ $\Rightarrow t^2\geq \frac{1}{4}$

Sử dụng BĐT dạng $(1)$: $\frac{t^2}{4}\geq x^2y^2$

Biến đổi: $P=3\left [t^2-x^2y^2 \right ]-2t+1\geq 3.\frac{3t^2}{4}-2t+1$

$=2(t-\frac{1}{2})^2+\frac{t^2}{4}+\frac{1}{2}\geq \frac{1}{16}+\frac{1}{2}=\frac{9}{16}$

 Vậy $MinP=\frac{9}{16}$  khi $x=y=\frac{1}{2}$

 

a,b chưa chắc dương bạn à   -_-




#485772 Trận 4 - Bất đẳng thức

Đã gửi bởi Simpson Joe Donald on 04-03-2014 - 11:39 trong Thi giải toán Marathon cấp THCS 2014

 

MSS 59

Ta có $(x+y)^{2}-4xy\geq 0$ (1)

         $(x+y)^{3}+4xy\geq 2$ (2)

Cộng (1) và (2) ta được $(x+y)^{3}+(x+y)^{2}-2\geq 0$

Đặt $t=(x+y)$

$\Rightarrow t^{3}+t^{2}-2\geq 0$

$\Leftrightarrow t^{3}+2t^{2}+2t-t^{2}-2t-2\geq 0$

$\Leftrightarrow t(t^{2}+2t+2)-(t^{2}+2t+2)\geq 0$

$\Leftrightarrow (t-1)(t^{2}+2t+2)\geq 0$

Mà $t^{2}+2t+2=t^{2}+2t+1+1=(t+1)^{2}+1\geq 1> 0$

$\Rightarrow t-1\geq 0$

$\Leftrightarrow x+y\geq 1$

Mặt khác $(x-y)^{2}\geq 0$

$\Rightarrow 2x^{2}-x^{2}-2xy+2y^{2}-y^{2}\geq 0$

$\Leftrightarrow 2(x^{2}+y^{2})\geq (x+y)^{2}$

$\Leftrightarrow x^{2}+y^{2}\geq \frac{(x+y)^{2}}{2}\geq \frac{1}{2}$ (Dấu "=" xảy ra khi $x=y=\frac{1}{2}$)

Lại có $(x^{2}+y^{2})^{2}\geq 4x^{2}y^{2}\Rightarrow \frac{(x^{2}+y^{2})^{2}}{4}\geq x^{2}y^{2}$
$\Rightarrow P=3(x^{4}+y^{4}+x^{2}y^{2})-2(x^{2}+y^{2})+1$
         $=3[(x^{2}+y^{2})^{2}-x^{2}y^{2}]-2(x^{2}+y^{2})+1$
         $\geq 3[(x^{2}+y^{2})^{2}-\frac{(x^{2}+y^{2})^{2}}{4}]-2(x^{2}+y^{2})+1$
         $\geq \frac{9}{4}(x^{2}+y^{2})^{2}-2(x^{2}+y^{2})+1$
Đặt $a=x^{2}+y^{2}$ (Điều kiện $a\geq \frac{1}{2}$)
$f(a)= \frac{9}{4}a^{2}-2a+1,a\geq \frac{1}{2}$
$f(a)\geq f(\frac{1}{2})=\frac{9}{16}$
Vậy $P_{min}=\frac{9}{16}\Leftrightarrow x=y=\frac{1}{2}$

 

2 lời giải này có lẽ không phù hợp với THCS, dùng cả đạo hàm :(




#485226 Trận 4 - Bất đẳng thức

Đã gửi bởi Simpson Joe Donald on 28-02-2014 - 22:44 trong Thi giải toán Marathon cấp THCS 2014

Mở rộng: Cho $x,y,a,b,c,k\in \mathbb{R}; \ a,b,k\neq 0$ thỏa mãn $(x+y)^3+4xy \ge 2$ . Tìm giá trị nhỏ nhất của: $$P=a\left(x^{2k}+y^{2k}+x^ky^k\right)−b\left(x^k+y^k\right)+c$$

Giải: 

Đầu tiên ta có bất đẳng thức cơ bản: $(a+b)^2\ge 4ab\iff (a-b)^2\ge 0$(luôn đúng). Dấu bằng xảy ra khi $a=b$

Đặt $x+y=t$, ta có: $$t^3+t^2=(x+y)^3+(x+y)^2\ge (x+y)^3+4xy\ge 2 \\ \implies t^3+t^2\ge 2$$

Xét bất phương trình: $$t^3+t^2-2\ge 0\iff (t-1)(t^2+2t+2)\ge 0\iff t\ge 1$$

Ta có: $x^{2k}+y^{2k}+x^ky^k\ge \dfrac{3}{4}\left(x^k+y^k\right)^2\iff \left(x^k-y^k\right)^2$ ( Luôn đúng)

Do đó: $$P\ge \dfrac{3a}{4}\left(x^k+y^k\right)^2−b\left(x^k+y^k\right)+c$$

Theo BĐT $Holder$ thì : $\left(x^k+y^k\right).2^{k-1}\ge (x+y)^k\implies x^k+y^k\ge \dfrac{2}{2^k}$

Đặt $x^k+y^k=m, \ m\ge \dfrac{2}{2^k}$, thì ta đưa về bài toán:

Tìm giá trị nhỏ nhất của $P=\dfrac{3am^2}{4}-bm+c$ với $m\ge \dfrac{2}{2^k}$

Đến đây dễ dàng tìm $\min P.$

 

Mở rộng thiếu hợp lí vì nếu các biến âm thì không tồn tại $x^k$, $y^k$ .




#485210 Trận 4 - Bất đẳng thức

Đã gửi bởi Simpson Joe Donald on 28-02-2014 - 22:04 trong Thi giải toán Marathon cấp THCS 2014

Đầu tiên ta có bất đẳng thức cơ bản:  $(a+b)^2\ge 4ab\iff (a-b)^2\ge 0$(luôn đúng). Dấu bằng xảy ra khi $a=b$

Đặt $x+y=t$, ta có: 

$$t^3+t^2=(x+y)^3+(x+y)^2\ge (x+y)^3+4xy\ge 2 \\ \implies t^3+t^2\ge 2$$

Xét bất phương trình: $$t^3+t^2-t\ge 0\iff (t-1)(t^2+2t+2)\ge 0\iff t\ge 1$$

Lại có: $x^4+y^4+x^2y^2\ge \dfrac{3}{4}(x^2+y^2)^2\iff (x^2-y^2)^2\ge 0$ ( luôn đúng)

Do đó: $$P\ge \dfrac{9}{4}(x^2+y^2)^2-2(x^2+y^2)+1$$

Đặt $x^2+y^2=k, \ k\ge 0$. Mà $2(x^2+y^2)\ge (x+y)^2\iff (x-y)^2\ge 0\implies 2k\ge t\implies k\ge \dfrac{1}{2}$

Ta đưa về bài toán, tìm min của: $P=\dfrac{9k^2}{4}-2k+1$ với $k\ge \dfrac{1}{2}$

Ta có: $$P=\dfrac{1}{4}(9k^2-8k+4)=\dfrac{1}{4}\left[ \left(k^2-\dfrac{1}{4}\right)+8\left(k-\dfrac{1}{2}\right)^2+\dfrac{9}{4}\right]=\dfrac{1}{4}\left[ \left(k^2-\dfrac{1}{4}\right)+8\left(k-\dfrac{1}{2}\right)^2\right]+\dfrac{9}{16}\ge \dfrac{9}{16}$$

Dấu bằng xảy ra khi $\begin{cases}k^2-\dfrac{1}{4}=0 \\ k-\dfrac{1}{2}=0\end{cases}\iff k=\dfrac{1}{2}\implies t=1\implies \begin{cases}x+y=1 \\ x=y\end{cases}\iff x=y=\dfrac{1}{2}$

Vậy $\min P=\dfrac{9}{16}$ khi và chỉ khi $x=y=\dfrac{1}{2}.\blacksquare$

 

Bỏ




#485219 Trận 4 - Bất đẳng thức

Đã gửi bởi Simpson Joe Donald on 28-02-2014 - 22:23 trong Thi giải toán Marathon cấp THCS 2014

Bài trên em nhầm chút, em xin làm lại:

Đầu tiên ta có bất đẳng thức cơ bản: $(a+b)^2\ge 4ab\iff (a-b)^2\ge 0$(luôn đúng). Dấu bằng xảy ra khi $a=b$

Đặt $x+y=t$, ta có: $$t^3+t^2=(x+y)^3+(x+y)^2\ge (x+y)^3+4xy\ge 2 \\ \implies t^3+t^2\ge 2$$

Xét bất phương trình: $$t^3+t^2-2\ge 0\iff (t-1)(t^2+2t+2)\ge 0\iff t\ge 1$$

Lại có: $x^4+y^4+x^2y^2\ge \dfrac{3}{4}(x^2+y^2)^2\iff (x^2-y^2)^2\ge 0$ ( luôn đúng)

Do đó: $$P\ge \dfrac{9}{4}(x^2+y^2)^2-2(x^2+y^2)+1$$

Đặt $x^2+y^2=k, \ k\ge 0$. Mà $2(x^2+y^2)\ge (x+y)^2\iff (x-y)^2\ge 0\implies 2k\ge t^2\implies k\ge \dfrac{1}{2}$

Ta đưa về bài toán, tìm min của: $P=\dfrac{9k^2}{4}-2k+1$ với $k\ge \dfrac{1}{2}$ Ta có: $$P=\dfrac{1}{4}(9k^2-8k+4)=\dfrac{1}{4}\left[ \left(k^2-\dfrac{1}{4}\right)+8\left(k-\dfrac{1}{2}\right)^2+\dfrac{9}{4}\right]=\dfrac{1}{4}\left[ \left(k^2-\dfrac{1}{4}\right)+8\left(k-\dfrac{1}{2}\right)^2\right]+\dfrac{9}{16}\ge \dfrac{9}{16}$$

Dấu bằng xảy ra khi $\begin{cases}k^2-\dfrac{1}{4}=0 \\ k-\dfrac{1}{2}=0\end{cases}\iff k=\dfrac{1}{2}\implies t=1\implies \begin{cases}x+y=1 \\ x=y\end{cases}\iff x=y=\dfrac{1}{2}$

Vậy $\min P=\dfrac{9}{16}$ khi và chỉ khi $x=y=\dfrac{1}{2}.\blacksquare$

 

Điểm 10.




#485765 Trận 4 - Bất đẳng thức

Đã gửi bởi Simpson Joe Donald on 04-03-2014 - 10:57 trong Thi giải toán Marathon cấp THCS 2014

Theo giả thiết ta có : $(x+y)^3+4xy\geq 2\Rightarrow 2\leq (x+y)^3+(x+y)^2\Leftrightarrow (x+y)^3+(x+y)^2-2\geq 0\Leftrightarrow \left ( x+y-1 \right )\left [ \left ( x+y \right )^2+x+y+2 \right ]\geq 0\Rightarrow x+y\geq 1$

Ta có : $P=3\left ( x^4+y^4+x^2y^2 \right )-2\left ( x^2+y^2 \right )+1$

Áp dụng BĐT $AM-GM$ với các số thực $a,b$ ta có : $4ab\leq \left ( a+b \right )^2,a^2+b^2\geq 2\left | ab \right |\geq 2ab$

$\Rightarrow 2P=6\left ( x^4+y^4+x^2y^2 \right )-4\left ( x^2+y^2 \right )+2=2\left ( x^4+y^4-x^2y^2 \right )+4\left ( x^4+y^4+2x^2y^2 \right )-4\left ( x^2+y^2 \right )+2=2\left ( x^4+y^4-x^2y^2 \right )+\left [ 2\left ( x^2+y^2 \right )-1 \right ]+1$

$\geq \left ( x^4+y^4 \right )+\left ( x^4+y^4-2x^2y^2 \right )+\left [ \left ( x+y \right )^2 \right ]+1\geq x^4+y^4+1\geq 2.\left ( \frac{x+y}{4} \right )^4+1\geq \frac{9}{8}\Rightarrow P\geq \frac{9}{16}$

Dấu = xảy ra khi $x=y=\frac{1}{2}$

$a,b$ có dương đâu mà $AM-GM$ bạn :P




#485288 Báo lỗi diễn đàn

Đã gửi bởi Simpson Joe Donald on 01-03-2014 - 17:38 trong Hướng dẫn - Trợ giúp - Giải đáp thắc mắc khi sử dụng Diễn đàn

http://diendantoanho...ức/#entry485174

 

Chủ đề này em không trích dẫn được ạ, em thử mấy lần r mà vẫn không được :(




#475129 Trận 1 - Phương trình nghiệm nguyên ...

Đã gửi bởi Simpson Joe Donald on 03-01-2014 - 22:44 trong Thi giải toán Marathon cấp THCS 2014

Lời giải: - MSS 33

 

Nhận thấy với $N=0 \ ; \ N=1$ thì hiển nhiên phương trình vô nghiệm.

Giả xử với $m\neq 1$ tồn tại $n,N,k$ sao cho phương trình (1) có nghiệm.

Với $m=0$ thì $N^m=1\implies \begin{cases}n^2+1=1\\44n^3+11n^2+10n+2=1\end{cases}$

hệ vô nghiệm nên $m=0$ không thỏa mãn phương trình , suy ra $m\ge 2$ (đến đây xét $m\ge 2$)

 

Xét $n=0$ phương trình vô nghiệm, với $n\ge 1$ thì: 

+ $TH_1: \ \ n=2k$ , ta có: $44n^3+11n^2+10n+2=2(176k^3+22n^2+10k+1)\vdots 2\implies N\vdots 2\implies N=2x$ 

tức là VP có dạng lũy thừa của 2. Mà $n^2+1=4k^2+1$ không chia hết cho 2.

Từ đó suy ra với $n=2k$ thì phương trình vô nghiệm.

+ $TH_2: \ \ n=2k+1$, ta có: $n^2+1=4k^2+4k+2$ chia hết cho 2 suy ra $N=2y$.

$44n^3+11n^2+10n+2$ không chia hết cho 2. Từ đó suy ra phương trình vô nghiệm với $n=2k+1$

Tóm lại phương trình vô nghiệm với mọi $m\neq 1$.

Với $m=1$ thì phương trình vô số nghiệm $n;N;k$

 

Kết luận: Với mọi số nguyên không âm $n;N;k$ thỏa mãn $(n^2+1)^{2^k}\cdot(44n^3+11n^2+10n+2)=N^m$ thì $m=1$

 

 

Điểm bài : 8 đ

 

 

Chỗ in đậm trường hợp 1 không đúng, như thế là ta đã nói rằng $N^m=2^l$.
Chỗ trường hợp 2, như lời giải trên đã chỉ rõ ràng cả $VT$ lẫn $VP$ đều chia hết cho $2$, vậy điều khiến cho phương trình vô nghiệm là ở chỗ nào ? (chỗ này cần chỉ rõ rằng $VT \equiv 2 \pmod{4}$ còn $VP \equiv 0 \pmod{4}$).

 

S = 16.3 + 3*8 = 40.3